Q13

 
GeneW
Thanks Received: 0
Jackie Chiles
Jackie Chiles
 
Posts: 45
Joined: October 11th, 2012
 
 
 

Q13

by GeneW Fri Nov 15, 2013 6:52 pm

Why is C a better answer than E? I am a bit puzzled. They both have JM. The only difference is F and G. Can someone please explain the difference.

Thank you in advance.
 
GeneW
Thanks Received: 0
Jackie Chiles
Jackie Chiles
 
Posts: 45
Joined: October 11th, 2012
 
 
 

Re: Q13

by GeneW Fri Nov 15, 2013 6:56 pm

No problem. I got it. E violates one of the rules. Thank you.
 
KelseyM440
Thanks Received: 0
Vinny Gambini
Vinny Gambini
 
Posts: 1
Joined: February 26th, 2018
 
 
 

Re: Q13

by KelseyM440 Tue Jul 03, 2018 11:21 am

I'm still confused about why C is the correct answer. I thought if F then Q so how can F be selected without Q? I might be confused about the general wording of the question.
User avatar
 
ohthatpatrick
Thanks Received: 3805
Atticus Finch
Atticus Finch
 
Posts: 4661
Joined: April 01st, 2011
 
 
 

Re: Q13

by ohthatpatrick Thu Jul 05, 2018 7:45 pm

This question is asking, "which of these answer choices contains three people who could be among the five selected?"

For (A), if we have FGJ all in, we also know that Q is in (Rule 3), K is in (Rule 4), M is in (Rule 5) and therefore P is out (Rule 6).

F G H K M Q | P _ _

This is broken because there are six people in.

--------------

For (B), if we have FGM all in, we also know that Q is in (Rule 3), K is in (Rule 4) and J is in (Rule 5) and P is out (Rule 6).

F G M K J Q | P _ _ _

Same problem as last time. 6 people IN.

---------------

For (C), if we have FJM all in, we know that Q is in (Rule 3) and P is out (Rule 6).

F J M Q __ | P __ __ __

This should work. We have three tenants so we need to pick our 5th person as a homeowner (from R and S). Either that person, R/S or Q would be chairperson. We should be good.

F J M Q R/S | P S/R G K

Correct Answer.


--------------

For (D), if we have GJK, then we also know we have M (rule 5) and thus don't have P (rule 6).

G J K M __ | P __ __ __

This is broken because it has four tenants, but we're only allowed a max of three.

-------------

For (E), if we have GJM in, then we know that K is in (rule 4) and P is out (rule 6).

G J M K __ | P __ __ __

This is broken just like (D), because it has FOUR tenants.

Hope this helps.